Is the direction of the electric field relative to the magnetic field in an electromagnetic wave a...












12












$begingroup$


In an electromagnetic wave.



EM wave



Could the magnetic field be mirrored around the xy-plane? Is there a specific reason that the 2 fields are oriented this way? Is it just a convention?










share|cite|improve this question











$endgroup$








  • 5




    $begingroup$
    Please, report the source of the picture.
    $endgroup$
    – Massimo Ortolano
    Jan 2 at 20:12










  • $begingroup$
    I am not sure if you are implying physical space, but the graph is not meant to imply the physical 3D x,y,z. The graph is a portrayal of vectors, mirroring would change values of vectors by negative or positive.
    $endgroup$
    – takintoolong
    Jan 3 at 3:46
















12












$begingroup$


In an electromagnetic wave.



EM wave



Could the magnetic field be mirrored around the xy-plane? Is there a specific reason that the 2 fields are oriented this way? Is it just a convention?










share|cite|improve this question











$endgroup$








  • 5




    $begingroup$
    Please, report the source of the picture.
    $endgroup$
    – Massimo Ortolano
    Jan 2 at 20:12










  • $begingroup$
    I am not sure if you are implying physical space, but the graph is not meant to imply the physical 3D x,y,z. The graph is a portrayal of vectors, mirroring would change values of vectors by negative or positive.
    $endgroup$
    – takintoolong
    Jan 3 at 3:46














12












12








12


4



$begingroup$


In an electromagnetic wave.



EM wave



Could the magnetic field be mirrored around the xy-plane? Is there a specific reason that the 2 fields are oriented this way? Is it just a convention?










share|cite|improve this question











$endgroup$




In an electromagnetic wave.



EM wave



Could the magnetic field be mirrored around the xy-plane? Is there a specific reason that the 2 fields are oriented this way? Is it just a convention?







electromagnetism waves electromagnetic-radiation polarization






share|cite|improve this question















share|cite|improve this question













share|cite|improve this question




share|cite|improve this question








edited Jan 2 at 20:07









Qmechanic

102k121831169




102k121831169










asked Jan 2 at 19:04









Alexander AmeyeAlexander Ameye

636




636








  • 5




    $begingroup$
    Please, report the source of the picture.
    $endgroup$
    – Massimo Ortolano
    Jan 2 at 20:12










  • $begingroup$
    I am not sure if you are implying physical space, but the graph is not meant to imply the physical 3D x,y,z. The graph is a portrayal of vectors, mirroring would change values of vectors by negative or positive.
    $endgroup$
    – takintoolong
    Jan 3 at 3:46














  • 5




    $begingroup$
    Please, report the source of the picture.
    $endgroup$
    – Massimo Ortolano
    Jan 2 at 20:12










  • $begingroup$
    I am not sure if you are implying physical space, but the graph is not meant to imply the physical 3D x,y,z. The graph is a portrayal of vectors, mirroring would change values of vectors by negative or positive.
    $endgroup$
    – takintoolong
    Jan 3 at 3:46








5




5




$begingroup$
Please, report the source of the picture.
$endgroup$
– Massimo Ortolano
Jan 2 at 20:12




$begingroup$
Please, report the source of the picture.
$endgroup$
– Massimo Ortolano
Jan 2 at 20:12












$begingroup$
I am not sure if you are implying physical space, but the graph is not meant to imply the physical 3D x,y,z. The graph is a portrayal of vectors, mirroring would change values of vectors by negative or positive.
$endgroup$
– takintoolong
Jan 3 at 3:46




$begingroup$
I am not sure if you are implying physical space, but the graph is not meant to imply the physical 3D x,y,z. The graph is a portrayal of vectors, mirroring would change values of vectors by negative or positive.
$endgroup$
– takintoolong
Jan 3 at 3:46










5 Answers
5






active

oldest

votes


















18












$begingroup$

If you were to flip the direction of the magnetic field in the above picture, it would describe light propagating in the opposite direction—so no, in this way, its direction is not a convention.



However, it's important to keep in mind that the direction of the magnetic field is a convention at a more fundamental level. It represents an oriented plane perpendicular to its direction via an orientation rule. If we all decided to use the "left hand rule" instead of the right hand rule for the cross product, then the magnetic field would point in the opposite direction. It is a bivector, or a pseudovector, field.



The electric field is a true vector field. At least in the three-dimensional description, which is itself a (very decent) convention, as long as we acknowledge that this picture depends on our reference frame.



In the four-dimensional picture, electric and magnetic fields can be understood as a single bivector field, such that a choice of reference frame determines a splitting of this bivector into independent "timelike" (electric) and "spacelike" (magnetic) planes.



That may be more than you were originally looking for, but I think this question makes for a good entry point to many of the other choices of representation that we make in physics.






share|cite|improve this answer











$endgroup$









  • 2




    $begingroup$
    Thanks for pointing out that the direction of the magnetic field is an arbitrary convention, but at a deeper level than the question is asking about.
    $endgroup$
    – Buzz
    Jan 3 at 2:56






  • 4




    $begingroup$
    The direction of electric field also a convention, just at a deeper level (i.e. signs assigned to kinds of charges).
    $endgroup$
    – Džuris
    Jan 3 at 11:48





















9












$begingroup$

The direction of the fields of an electromagnetic wave is not conventional but it's determined by Maxwell's equations. For the particular case of a plane electromagnetic wave of angular frequency $omega$ in vacuum, which can be represented by the picture you posted, Maxwell's equations require that the wave vector $boldsymbol{k}$, the electric field $boldsymbol{E}$ and the magnetic flux density $boldsymbol{B}$ obey the relationships (the dot represent the scalar product and the cross the vector product)



$$begin{align}&boldsymbol{k}cdot boldsymbol{E} = 0, \ &boldsymbol{k}cdot boldsymbol{B} = 0, \ & boldsymbol{B}=frac{1}{omega}boldsymbol{k}times boldsymbol{E}.end{align}$$



This means that $boldsymbol{k}$, $boldsymbol{E}$ and $boldsymbol{B}$ are three orthogonal vectors and that the direction of any one of them is determined by the other two. Therefore, no, you cannot mirror the magnetic field in the picture.



Note that if it were possible to mirror just the magnetic field, then, by superposition, you would be able to construct a wave having nonzero electric field but zero magnetic field.






share|cite|improve this answer











$endgroup$









  • 1




    $begingroup$
    You may want to start with "Having defined the direction of B by the Lorentz rule ... " (BTW -1 was not me).
    $endgroup$
    – Bruce Greetham
    Jan 2 at 20:17






  • 3




    $begingroup$
    @BruceGreetham The point is that the magnetic field is defined in that way: once you have a certain definition, you cannot mirror the magnetic field (which is, I think, what the OP is asking).
    $endgroup$
    – Massimo Ortolano
    Jan 2 at 20:21








  • 2




    $begingroup$
    The third equation you give is written in terms of the cross product, and the direction of the cross product (right hand rule) is due to convention.
    $endgroup$
    – Acccumulation
    Jan 2 at 23:06










  • $begingroup$
    @Acccumulation I know: see my comment above to Bruce.
    $endgroup$
    – Massimo Ortolano
    Jan 2 at 23:10



















5












$begingroup$

The arrows are pointing in the "positive" direction of the fields. Which direction is "positive" is arbitrary (you can say that the "positive" direction in an electrical field in the direction in which the electrical potential energy for a positively charged particle is increasing, but what charge is "positive" is itself a convention). Moreover, those conventions are chosen independently. However, once those conventions are chosen, the directions of the fields for an electromagnetic wave are fixed. So if you're asking whether the conventions could have been chosen such that the magnetic field would be flipped, the answer is "yes". If you're asking whether we can flip the magnetic field, without affecting any of the other conventions in physics, the answer is "no".






share|cite|improve this answer









$endgroup$





















    3












    $begingroup$

    B is defined by the expression of the Lorentz force. In covariant notation, this is written as $f^nu = j_mu F^{munu}$, where



    ${displaystyle F^{mu nu }={begin{bmatrix}0&-E_{x}/c&-E_{y}/c&-E_{z}/c\E_{x}/c&0&-B_{z}&B_{y}\E_{y}/c&B_{z}&0&-B_{x}\E_{z}/c&-B_{y}&B_{x}&0end{bmatrix}}.} $



    One can see this as defining the Lorentz force as $vec f = q vec E + q vec v times vec B$, as well as $vec E$ and $vec B$.



    One can replace B by -B, or E by -E. This would be useless as the measurable physical quantities such as forces and energies would not change. Students would have to learn cumbersome definitions like like "$vec E$ is the opposite of the force per charge".






    share|cite|improve this answer











    $endgroup$













    • $begingroup$
      Well, one could define charge with an opposite sign as well, and so on.
      $endgroup$
      – rubenvb
      Jan 3 at 11:44



















    -5












    $begingroup$

    Electrons in an antenna rod, accelerated in one direction, all emit photons with the same spin of their electric and magnetic field components. This is obvious, since the receiving antennas could be made of a rod to receive the changing electric field or could be made of a ring (available in the German wiki only) to receive the magnetic field. If accelerated in the same direction electrons would have spin clockwise and anti clockwise one would not be able to use the magnetic component (because both variations will cancel out each over).



    I’m not sure but think that protons and positrons will have the opposite spin to electrons and antiprotons. Clarification from specialists will be nice.






    share|cite|improve this answer









    $endgroup$









    • 4




      $begingroup$
      This question is not related to spin.
      $endgroup$
      – my2cts
      Jan 2 at 21:17






    • 2




      $begingroup$
      To the extent that all questions about electromagnetism involve spin, this answer is still quite wrong. The radiation from a rod antenna is linearly polarized. The photons which make up linearly-polarized radiation are in a superposition of left- and right-handed spin states. The sign of the more mobile charges in the antenna does not contribute.
      $endgroup$
      – rob
      Jan 3 at 0:59













    Your Answer





    StackExchange.ifUsing("editor", function () {
    return StackExchange.using("mathjaxEditing", function () {
    StackExchange.MarkdownEditor.creationCallbacks.add(function (editor, postfix) {
    StackExchange.mathjaxEditing.prepareWmdForMathJax(editor, postfix, [["$", "$"], ["\\(","\\)"]]);
    });
    });
    }, "mathjax-editing");

    StackExchange.ready(function() {
    var channelOptions = {
    tags: "".split(" "),
    id: "151"
    };
    initTagRenderer("".split(" "), "".split(" "), channelOptions);

    StackExchange.using("externalEditor", function() {
    // Have to fire editor after snippets, if snippets enabled
    if (StackExchange.settings.snippets.snippetsEnabled) {
    StackExchange.using("snippets", function() {
    createEditor();
    });
    }
    else {
    createEditor();
    }
    });

    function createEditor() {
    StackExchange.prepareEditor({
    heartbeatType: 'answer',
    autoActivateHeartbeat: false,
    convertImagesToLinks: false,
    noModals: true,
    showLowRepImageUploadWarning: true,
    reputationToPostImages: null,
    bindNavPrevention: true,
    postfix: "",
    imageUploader: {
    brandingHtml: "Powered by u003ca class="icon-imgur-white" href="https://imgur.com/"u003eu003c/au003e",
    contentPolicyHtml: "User contributions licensed under u003ca href="https://creativecommons.org/licenses/by-sa/3.0/"u003ecc by-sa 3.0 with attribution requiredu003c/au003e u003ca href="https://stackoverflow.com/legal/content-policy"u003e(content policy)u003c/au003e",
    allowUrls: true
    },
    noCode: true, onDemand: true,
    discardSelector: ".discard-answer"
    ,immediatelyShowMarkdownHelp:true
    });


    }
    });














    draft saved

    draft discarded


















    StackExchange.ready(
    function () {
    StackExchange.openid.initPostLogin('.new-post-login', 'https%3a%2f%2fphysics.stackexchange.com%2fquestions%2f451751%2fis-the-direction-of-the-electric-field-relative-to-the-magnetic-field-in-an-elec%23new-answer', 'question_page');
    }
    );

    Post as a guest















    Required, but never shown

























    5 Answers
    5






    active

    oldest

    votes








    5 Answers
    5






    active

    oldest

    votes









    active

    oldest

    votes






    active

    oldest

    votes









    18












    $begingroup$

    If you were to flip the direction of the magnetic field in the above picture, it would describe light propagating in the opposite direction—so no, in this way, its direction is not a convention.



    However, it's important to keep in mind that the direction of the magnetic field is a convention at a more fundamental level. It represents an oriented plane perpendicular to its direction via an orientation rule. If we all decided to use the "left hand rule" instead of the right hand rule for the cross product, then the magnetic field would point in the opposite direction. It is a bivector, or a pseudovector, field.



    The electric field is a true vector field. At least in the three-dimensional description, which is itself a (very decent) convention, as long as we acknowledge that this picture depends on our reference frame.



    In the four-dimensional picture, electric and magnetic fields can be understood as a single bivector field, such that a choice of reference frame determines a splitting of this bivector into independent "timelike" (electric) and "spacelike" (magnetic) planes.



    That may be more than you were originally looking for, but I think this question makes for a good entry point to many of the other choices of representation that we make in physics.






    share|cite|improve this answer











    $endgroup$









    • 2




      $begingroup$
      Thanks for pointing out that the direction of the magnetic field is an arbitrary convention, but at a deeper level than the question is asking about.
      $endgroup$
      – Buzz
      Jan 3 at 2:56






    • 4




      $begingroup$
      The direction of electric field also a convention, just at a deeper level (i.e. signs assigned to kinds of charges).
      $endgroup$
      – Džuris
      Jan 3 at 11:48


















    18












    $begingroup$

    If you were to flip the direction of the magnetic field in the above picture, it would describe light propagating in the opposite direction—so no, in this way, its direction is not a convention.



    However, it's important to keep in mind that the direction of the magnetic field is a convention at a more fundamental level. It represents an oriented plane perpendicular to its direction via an orientation rule. If we all decided to use the "left hand rule" instead of the right hand rule for the cross product, then the magnetic field would point in the opposite direction. It is a bivector, or a pseudovector, field.



    The electric field is a true vector field. At least in the three-dimensional description, which is itself a (very decent) convention, as long as we acknowledge that this picture depends on our reference frame.



    In the four-dimensional picture, electric and magnetic fields can be understood as a single bivector field, such that a choice of reference frame determines a splitting of this bivector into independent "timelike" (electric) and "spacelike" (magnetic) planes.



    That may be more than you were originally looking for, but I think this question makes for a good entry point to many of the other choices of representation that we make in physics.






    share|cite|improve this answer











    $endgroup$









    • 2




      $begingroup$
      Thanks for pointing out that the direction of the magnetic field is an arbitrary convention, but at a deeper level than the question is asking about.
      $endgroup$
      – Buzz
      Jan 3 at 2:56






    • 4




      $begingroup$
      The direction of electric field also a convention, just at a deeper level (i.e. signs assigned to kinds of charges).
      $endgroup$
      – Džuris
      Jan 3 at 11:48
















    18












    18








    18





    $begingroup$

    If you were to flip the direction of the magnetic field in the above picture, it would describe light propagating in the opposite direction—so no, in this way, its direction is not a convention.



    However, it's important to keep in mind that the direction of the magnetic field is a convention at a more fundamental level. It represents an oriented plane perpendicular to its direction via an orientation rule. If we all decided to use the "left hand rule" instead of the right hand rule for the cross product, then the magnetic field would point in the opposite direction. It is a bivector, or a pseudovector, field.



    The electric field is a true vector field. At least in the three-dimensional description, which is itself a (very decent) convention, as long as we acknowledge that this picture depends on our reference frame.



    In the four-dimensional picture, electric and magnetic fields can be understood as a single bivector field, such that a choice of reference frame determines a splitting of this bivector into independent "timelike" (electric) and "spacelike" (magnetic) planes.



    That may be more than you were originally looking for, but I think this question makes for a good entry point to many of the other choices of representation that we make in physics.






    share|cite|improve this answer











    $endgroup$



    If you were to flip the direction of the magnetic field in the above picture, it would describe light propagating in the opposite direction—so no, in this way, its direction is not a convention.



    However, it's important to keep in mind that the direction of the magnetic field is a convention at a more fundamental level. It represents an oriented plane perpendicular to its direction via an orientation rule. If we all decided to use the "left hand rule" instead of the right hand rule for the cross product, then the magnetic field would point in the opposite direction. It is a bivector, or a pseudovector, field.



    The electric field is a true vector field. At least in the three-dimensional description, which is itself a (very decent) convention, as long as we acknowledge that this picture depends on our reference frame.



    In the four-dimensional picture, electric and magnetic fields can be understood as a single bivector field, such that a choice of reference frame determines a splitting of this bivector into independent "timelike" (electric) and "spacelike" (magnetic) planes.



    That may be more than you were originally looking for, but I think this question makes for a good entry point to many of the other choices of representation that we make in physics.







    share|cite|improve this answer














    share|cite|improve this answer



    share|cite|improve this answer








    edited Jan 2 at 21:29

























    answered Jan 2 at 20:14









    Luke BurnsLuke Burns

    459616




    459616








    • 2




      $begingroup$
      Thanks for pointing out that the direction of the magnetic field is an arbitrary convention, but at a deeper level than the question is asking about.
      $endgroup$
      – Buzz
      Jan 3 at 2:56






    • 4




      $begingroup$
      The direction of electric field also a convention, just at a deeper level (i.e. signs assigned to kinds of charges).
      $endgroup$
      – Džuris
      Jan 3 at 11:48
















    • 2




      $begingroup$
      Thanks for pointing out that the direction of the magnetic field is an arbitrary convention, but at a deeper level than the question is asking about.
      $endgroup$
      – Buzz
      Jan 3 at 2:56






    • 4




      $begingroup$
      The direction of electric field also a convention, just at a deeper level (i.e. signs assigned to kinds of charges).
      $endgroup$
      – Džuris
      Jan 3 at 11:48










    2




    2




    $begingroup$
    Thanks for pointing out that the direction of the magnetic field is an arbitrary convention, but at a deeper level than the question is asking about.
    $endgroup$
    – Buzz
    Jan 3 at 2:56




    $begingroup$
    Thanks for pointing out that the direction of the magnetic field is an arbitrary convention, but at a deeper level than the question is asking about.
    $endgroup$
    – Buzz
    Jan 3 at 2:56




    4




    4




    $begingroup$
    The direction of electric field also a convention, just at a deeper level (i.e. signs assigned to kinds of charges).
    $endgroup$
    – Džuris
    Jan 3 at 11:48






    $begingroup$
    The direction of electric field also a convention, just at a deeper level (i.e. signs assigned to kinds of charges).
    $endgroup$
    – Džuris
    Jan 3 at 11:48













    9












    $begingroup$

    The direction of the fields of an electromagnetic wave is not conventional but it's determined by Maxwell's equations. For the particular case of a plane electromagnetic wave of angular frequency $omega$ in vacuum, which can be represented by the picture you posted, Maxwell's equations require that the wave vector $boldsymbol{k}$, the electric field $boldsymbol{E}$ and the magnetic flux density $boldsymbol{B}$ obey the relationships (the dot represent the scalar product and the cross the vector product)



    $$begin{align}&boldsymbol{k}cdot boldsymbol{E} = 0, \ &boldsymbol{k}cdot boldsymbol{B} = 0, \ & boldsymbol{B}=frac{1}{omega}boldsymbol{k}times boldsymbol{E}.end{align}$$



    This means that $boldsymbol{k}$, $boldsymbol{E}$ and $boldsymbol{B}$ are three orthogonal vectors and that the direction of any one of them is determined by the other two. Therefore, no, you cannot mirror the magnetic field in the picture.



    Note that if it were possible to mirror just the magnetic field, then, by superposition, you would be able to construct a wave having nonzero electric field but zero magnetic field.






    share|cite|improve this answer











    $endgroup$









    • 1




      $begingroup$
      You may want to start with "Having defined the direction of B by the Lorentz rule ... " (BTW -1 was not me).
      $endgroup$
      – Bruce Greetham
      Jan 2 at 20:17






    • 3




      $begingroup$
      @BruceGreetham The point is that the magnetic field is defined in that way: once you have a certain definition, you cannot mirror the magnetic field (which is, I think, what the OP is asking).
      $endgroup$
      – Massimo Ortolano
      Jan 2 at 20:21








    • 2




      $begingroup$
      The third equation you give is written in terms of the cross product, and the direction of the cross product (right hand rule) is due to convention.
      $endgroup$
      – Acccumulation
      Jan 2 at 23:06










    • $begingroup$
      @Acccumulation I know: see my comment above to Bruce.
      $endgroup$
      – Massimo Ortolano
      Jan 2 at 23:10
















    9












    $begingroup$

    The direction of the fields of an electromagnetic wave is not conventional but it's determined by Maxwell's equations. For the particular case of a plane electromagnetic wave of angular frequency $omega$ in vacuum, which can be represented by the picture you posted, Maxwell's equations require that the wave vector $boldsymbol{k}$, the electric field $boldsymbol{E}$ and the magnetic flux density $boldsymbol{B}$ obey the relationships (the dot represent the scalar product and the cross the vector product)



    $$begin{align}&boldsymbol{k}cdot boldsymbol{E} = 0, \ &boldsymbol{k}cdot boldsymbol{B} = 0, \ & boldsymbol{B}=frac{1}{omega}boldsymbol{k}times boldsymbol{E}.end{align}$$



    This means that $boldsymbol{k}$, $boldsymbol{E}$ and $boldsymbol{B}$ are three orthogonal vectors and that the direction of any one of them is determined by the other two. Therefore, no, you cannot mirror the magnetic field in the picture.



    Note that if it were possible to mirror just the magnetic field, then, by superposition, you would be able to construct a wave having nonzero electric field but zero magnetic field.






    share|cite|improve this answer











    $endgroup$









    • 1




      $begingroup$
      You may want to start with "Having defined the direction of B by the Lorentz rule ... " (BTW -1 was not me).
      $endgroup$
      – Bruce Greetham
      Jan 2 at 20:17






    • 3




      $begingroup$
      @BruceGreetham The point is that the magnetic field is defined in that way: once you have a certain definition, you cannot mirror the magnetic field (which is, I think, what the OP is asking).
      $endgroup$
      – Massimo Ortolano
      Jan 2 at 20:21








    • 2




      $begingroup$
      The third equation you give is written in terms of the cross product, and the direction of the cross product (right hand rule) is due to convention.
      $endgroup$
      – Acccumulation
      Jan 2 at 23:06










    • $begingroup$
      @Acccumulation I know: see my comment above to Bruce.
      $endgroup$
      – Massimo Ortolano
      Jan 2 at 23:10














    9












    9








    9





    $begingroup$

    The direction of the fields of an electromagnetic wave is not conventional but it's determined by Maxwell's equations. For the particular case of a plane electromagnetic wave of angular frequency $omega$ in vacuum, which can be represented by the picture you posted, Maxwell's equations require that the wave vector $boldsymbol{k}$, the electric field $boldsymbol{E}$ and the magnetic flux density $boldsymbol{B}$ obey the relationships (the dot represent the scalar product and the cross the vector product)



    $$begin{align}&boldsymbol{k}cdot boldsymbol{E} = 0, \ &boldsymbol{k}cdot boldsymbol{B} = 0, \ & boldsymbol{B}=frac{1}{omega}boldsymbol{k}times boldsymbol{E}.end{align}$$



    This means that $boldsymbol{k}$, $boldsymbol{E}$ and $boldsymbol{B}$ are three orthogonal vectors and that the direction of any one of them is determined by the other two. Therefore, no, you cannot mirror the magnetic field in the picture.



    Note that if it were possible to mirror just the magnetic field, then, by superposition, you would be able to construct a wave having nonzero electric field but zero magnetic field.






    share|cite|improve this answer











    $endgroup$



    The direction of the fields of an electromagnetic wave is not conventional but it's determined by Maxwell's equations. For the particular case of a plane electromagnetic wave of angular frequency $omega$ in vacuum, which can be represented by the picture you posted, Maxwell's equations require that the wave vector $boldsymbol{k}$, the electric field $boldsymbol{E}$ and the magnetic flux density $boldsymbol{B}$ obey the relationships (the dot represent the scalar product and the cross the vector product)



    $$begin{align}&boldsymbol{k}cdot boldsymbol{E} = 0, \ &boldsymbol{k}cdot boldsymbol{B} = 0, \ & boldsymbol{B}=frac{1}{omega}boldsymbol{k}times boldsymbol{E}.end{align}$$



    This means that $boldsymbol{k}$, $boldsymbol{E}$ and $boldsymbol{B}$ are three orthogonal vectors and that the direction of any one of them is determined by the other two. Therefore, no, you cannot mirror the magnetic field in the picture.



    Note that if it were possible to mirror just the magnetic field, then, by superposition, you would be able to construct a wave having nonzero electric field but zero magnetic field.







    share|cite|improve this answer














    share|cite|improve this answer



    share|cite|improve this answer








    edited Jan 2 at 23:03

























    answered Jan 2 at 20:01









    Massimo OrtolanoMassimo Ortolano

    1,9451615




    1,9451615








    • 1




      $begingroup$
      You may want to start with "Having defined the direction of B by the Lorentz rule ... " (BTW -1 was not me).
      $endgroup$
      – Bruce Greetham
      Jan 2 at 20:17






    • 3




      $begingroup$
      @BruceGreetham The point is that the magnetic field is defined in that way: once you have a certain definition, you cannot mirror the magnetic field (which is, I think, what the OP is asking).
      $endgroup$
      – Massimo Ortolano
      Jan 2 at 20:21








    • 2




      $begingroup$
      The third equation you give is written in terms of the cross product, and the direction of the cross product (right hand rule) is due to convention.
      $endgroup$
      – Acccumulation
      Jan 2 at 23:06










    • $begingroup$
      @Acccumulation I know: see my comment above to Bruce.
      $endgroup$
      – Massimo Ortolano
      Jan 2 at 23:10














    • 1




      $begingroup$
      You may want to start with "Having defined the direction of B by the Lorentz rule ... " (BTW -1 was not me).
      $endgroup$
      – Bruce Greetham
      Jan 2 at 20:17






    • 3




      $begingroup$
      @BruceGreetham The point is that the magnetic field is defined in that way: once you have a certain definition, you cannot mirror the magnetic field (which is, I think, what the OP is asking).
      $endgroup$
      – Massimo Ortolano
      Jan 2 at 20:21








    • 2




      $begingroup$
      The third equation you give is written in terms of the cross product, and the direction of the cross product (right hand rule) is due to convention.
      $endgroup$
      – Acccumulation
      Jan 2 at 23:06










    • $begingroup$
      @Acccumulation I know: see my comment above to Bruce.
      $endgroup$
      – Massimo Ortolano
      Jan 2 at 23:10








    1




    1




    $begingroup$
    You may want to start with "Having defined the direction of B by the Lorentz rule ... " (BTW -1 was not me).
    $endgroup$
    – Bruce Greetham
    Jan 2 at 20:17




    $begingroup$
    You may want to start with "Having defined the direction of B by the Lorentz rule ... " (BTW -1 was not me).
    $endgroup$
    – Bruce Greetham
    Jan 2 at 20:17




    3




    3




    $begingroup$
    @BruceGreetham The point is that the magnetic field is defined in that way: once you have a certain definition, you cannot mirror the magnetic field (which is, I think, what the OP is asking).
    $endgroup$
    – Massimo Ortolano
    Jan 2 at 20:21






    $begingroup$
    @BruceGreetham The point is that the magnetic field is defined in that way: once you have a certain definition, you cannot mirror the magnetic field (which is, I think, what the OP is asking).
    $endgroup$
    – Massimo Ortolano
    Jan 2 at 20:21






    2




    2




    $begingroup$
    The third equation you give is written in terms of the cross product, and the direction of the cross product (right hand rule) is due to convention.
    $endgroup$
    – Acccumulation
    Jan 2 at 23:06




    $begingroup$
    The third equation you give is written in terms of the cross product, and the direction of the cross product (right hand rule) is due to convention.
    $endgroup$
    – Acccumulation
    Jan 2 at 23:06












    $begingroup$
    @Acccumulation I know: see my comment above to Bruce.
    $endgroup$
    – Massimo Ortolano
    Jan 2 at 23:10




    $begingroup$
    @Acccumulation I know: see my comment above to Bruce.
    $endgroup$
    – Massimo Ortolano
    Jan 2 at 23:10











    5












    $begingroup$

    The arrows are pointing in the "positive" direction of the fields. Which direction is "positive" is arbitrary (you can say that the "positive" direction in an electrical field in the direction in which the electrical potential energy for a positively charged particle is increasing, but what charge is "positive" is itself a convention). Moreover, those conventions are chosen independently. However, once those conventions are chosen, the directions of the fields for an electromagnetic wave are fixed. So if you're asking whether the conventions could have been chosen such that the magnetic field would be flipped, the answer is "yes". If you're asking whether we can flip the magnetic field, without affecting any of the other conventions in physics, the answer is "no".






    share|cite|improve this answer









    $endgroup$


















      5












      $begingroup$

      The arrows are pointing in the "positive" direction of the fields. Which direction is "positive" is arbitrary (you can say that the "positive" direction in an electrical field in the direction in which the electrical potential energy for a positively charged particle is increasing, but what charge is "positive" is itself a convention). Moreover, those conventions are chosen independently. However, once those conventions are chosen, the directions of the fields for an electromagnetic wave are fixed. So if you're asking whether the conventions could have been chosen such that the magnetic field would be flipped, the answer is "yes". If you're asking whether we can flip the magnetic field, without affecting any of the other conventions in physics, the answer is "no".






      share|cite|improve this answer









      $endgroup$
















        5












        5








        5





        $begingroup$

        The arrows are pointing in the "positive" direction of the fields. Which direction is "positive" is arbitrary (you can say that the "positive" direction in an electrical field in the direction in which the electrical potential energy for a positively charged particle is increasing, but what charge is "positive" is itself a convention). Moreover, those conventions are chosen independently. However, once those conventions are chosen, the directions of the fields for an electromagnetic wave are fixed. So if you're asking whether the conventions could have been chosen such that the magnetic field would be flipped, the answer is "yes". If you're asking whether we can flip the magnetic field, without affecting any of the other conventions in physics, the answer is "no".






        share|cite|improve this answer









        $endgroup$



        The arrows are pointing in the "positive" direction of the fields. Which direction is "positive" is arbitrary (you can say that the "positive" direction in an electrical field in the direction in which the electrical potential energy for a positively charged particle is increasing, but what charge is "positive" is itself a convention). Moreover, those conventions are chosen independently. However, once those conventions are chosen, the directions of the fields for an electromagnetic wave are fixed. So if you're asking whether the conventions could have been chosen such that the magnetic field would be flipped, the answer is "yes". If you're asking whether we can flip the magnetic field, without affecting any of the other conventions in physics, the answer is "no".







        share|cite|improve this answer












        share|cite|improve this answer



        share|cite|improve this answer










        answered Jan 2 at 23:13









        AcccumulationAcccumulation

        1,886210




        1,886210























            3












            $begingroup$

            B is defined by the expression of the Lorentz force. In covariant notation, this is written as $f^nu = j_mu F^{munu}$, where



            ${displaystyle F^{mu nu }={begin{bmatrix}0&-E_{x}/c&-E_{y}/c&-E_{z}/c\E_{x}/c&0&-B_{z}&B_{y}\E_{y}/c&B_{z}&0&-B_{x}\E_{z}/c&-B_{y}&B_{x}&0end{bmatrix}}.} $



            One can see this as defining the Lorentz force as $vec f = q vec E + q vec v times vec B$, as well as $vec E$ and $vec B$.



            One can replace B by -B, or E by -E. This would be useless as the measurable physical quantities such as forces and energies would not change. Students would have to learn cumbersome definitions like like "$vec E$ is the opposite of the force per charge".






            share|cite|improve this answer











            $endgroup$













            • $begingroup$
              Well, one could define charge with an opposite sign as well, and so on.
              $endgroup$
              – rubenvb
              Jan 3 at 11:44
















            3












            $begingroup$

            B is defined by the expression of the Lorentz force. In covariant notation, this is written as $f^nu = j_mu F^{munu}$, where



            ${displaystyle F^{mu nu }={begin{bmatrix}0&-E_{x}/c&-E_{y}/c&-E_{z}/c\E_{x}/c&0&-B_{z}&B_{y}\E_{y}/c&B_{z}&0&-B_{x}\E_{z}/c&-B_{y}&B_{x}&0end{bmatrix}}.} $



            One can see this as defining the Lorentz force as $vec f = q vec E + q vec v times vec B$, as well as $vec E$ and $vec B$.



            One can replace B by -B, or E by -E. This would be useless as the measurable physical quantities such as forces and energies would not change. Students would have to learn cumbersome definitions like like "$vec E$ is the opposite of the force per charge".






            share|cite|improve this answer











            $endgroup$













            • $begingroup$
              Well, one could define charge with an opposite sign as well, and so on.
              $endgroup$
              – rubenvb
              Jan 3 at 11:44














            3












            3








            3





            $begingroup$

            B is defined by the expression of the Lorentz force. In covariant notation, this is written as $f^nu = j_mu F^{munu}$, where



            ${displaystyle F^{mu nu }={begin{bmatrix}0&-E_{x}/c&-E_{y}/c&-E_{z}/c\E_{x}/c&0&-B_{z}&B_{y}\E_{y}/c&B_{z}&0&-B_{x}\E_{z}/c&-B_{y}&B_{x}&0end{bmatrix}}.} $



            One can see this as defining the Lorentz force as $vec f = q vec E + q vec v times vec B$, as well as $vec E$ and $vec B$.



            One can replace B by -B, or E by -E. This would be useless as the measurable physical quantities such as forces and energies would not change. Students would have to learn cumbersome definitions like like "$vec E$ is the opposite of the force per charge".






            share|cite|improve this answer











            $endgroup$



            B is defined by the expression of the Lorentz force. In covariant notation, this is written as $f^nu = j_mu F^{munu}$, where



            ${displaystyle F^{mu nu }={begin{bmatrix}0&-E_{x}/c&-E_{y}/c&-E_{z}/c\E_{x}/c&0&-B_{z}&B_{y}\E_{y}/c&B_{z}&0&-B_{x}\E_{z}/c&-B_{y}&B_{x}&0end{bmatrix}}.} $



            One can see this as defining the Lorentz force as $vec f = q vec E + q vec v times vec B$, as well as $vec E$ and $vec B$.



            One can replace B by -B, or E by -E. This would be useless as the measurable physical quantities such as forces and energies would not change. Students would have to learn cumbersome definitions like like "$vec E$ is the opposite of the force per charge".







            share|cite|improve this answer














            share|cite|improve this answer



            share|cite|improve this answer








            edited Jan 3 at 0:13

























            answered Jan 3 at 0:06









            my2ctsmy2cts

            4,7802618




            4,7802618












            • $begingroup$
              Well, one could define charge with an opposite sign as well, and so on.
              $endgroup$
              – rubenvb
              Jan 3 at 11:44


















            • $begingroup$
              Well, one could define charge with an opposite sign as well, and so on.
              $endgroup$
              – rubenvb
              Jan 3 at 11:44
















            $begingroup$
            Well, one could define charge with an opposite sign as well, and so on.
            $endgroup$
            – rubenvb
            Jan 3 at 11:44




            $begingroup$
            Well, one could define charge with an opposite sign as well, and so on.
            $endgroup$
            – rubenvb
            Jan 3 at 11:44











            -5












            $begingroup$

            Electrons in an antenna rod, accelerated in one direction, all emit photons with the same spin of their electric and magnetic field components. This is obvious, since the receiving antennas could be made of a rod to receive the changing electric field or could be made of a ring (available in the German wiki only) to receive the magnetic field. If accelerated in the same direction electrons would have spin clockwise and anti clockwise one would not be able to use the magnetic component (because both variations will cancel out each over).



            I’m not sure but think that protons and positrons will have the opposite spin to electrons and antiprotons. Clarification from specialists will be nice.






            share|cite|improve this answer









            $endgroup$









            • 4




              $begingroup$
              This question is not related to spin.
              $endgroup$
              – my2cts
              Jan 2 at 21:17






            • 2




              $begingroup$
              To the extent that all questions about electromagnetism involve spin, this answer is still quite wrong. The radiation from a rod antenna is linearly polarized. The photons which make up linearly-polarized radiation are in a superposition of left- and right-handed spin states. The sign of the more mobile charges in the antenna does not contribute.
              $endgroup$
              – rob
              Jan 3 at 0:59


















            -5












            $begingroup$

            Electrons in an antenna rod, accelerated in one direction, all emit photons with the same spin of their electric and magnetic field components. This is obvious, since the receiving antennas could be made of a rod to receive the changing electric field or could be made of a ring (available in the German wiki only) to receive the magnetic field. If accelerated in the same direction electrons would have spin clockwise and anti clockwise one would not be able to use the magnetic component (because both variations will cancel out each over).



            I’m not sure but think that protons and positrons will have the opposite spin to electrons and antiprotons. Clarification from specialists will be nice.






            share|cite|improve this answer









            $endgroup$









            • 4




              $begingroup$
              This question is not related to spin.
              $endgroup$
              – my2cts
              Jan 2 at 21:17






            • 2




              $begingroup$
              To the extent that all questions about electromagnetism involve spin, this answer is still quite wrong. The radiation from a rod antenna is linearly polarized. The photons which make up linearly-polarized radiation are in a superposition of left- and right-handed spin states. The sign of the more mobile charges in the antenna does not contribute.
              $endgroup$
              – rob
              Jan 3 at 0:59
















            -5












            -5








            -5





            $begingroup$

            Electrons in an antenna rod, accelerated in one direction, all emit photons with the same spin of their electric and magnetic field components. This is obvious, since the receiving antennas could be made of a rod to receive the changing electric field or could be made of a ring (available in the German wiki only) to receive the magnetic field. If accelerated in the same direction electrons would have spin clockwise and anti clockwise one would not be able to use the magnetic component (because both variations will cancel out each over).



            I’m not sure but think that protons and positrons will have the opposite spin to electrons and antiprotons. Clarification from specialists will be nice.






            share|cite|improve this answer









            $endgroup$



            Electrons in an antenna rod, accelerated in one direction, all emit photons with the same spin of their electric and magnetic field components. This is obvious, since the receiving antennas could be made of a rod to receive the changing electric field or could be made of a ring (available in the German wiki only) to receive the magnetic field. If accelerated in the same direction electrons would have spin clockwise and anti clockwise one would not be able to use the magnetic component (because both variations will cancel out each over).



            I’m not sure but think that protons and positrons will have the opposite spin to electrons and antiprotons. Clarification from specialists will be nice.







            share|cite|improve this answer












            share|cite|improve this answer



            share|cite|improve this answer










            answered Jan 2 at 19:27









            HolgerFiedlerHolgerFiedler

            4,15031134




            4,15031134








            • 4




              $begingroup$
              This question is not related to spin.
              $endgroup$
              – my2cts
              Jan 2 at 21:17






            • 2




              $begingroup$
              To the extent that all questions about electromagnetism involve spin, this answer is still quite wrong. The radiation from a rod antenna is linearly polarized. The photons which make up linearly-polarized radiation are in a superposition of left- and right-handed spin states. The sign of the more mobile charges in the antenna does not contribute.
              $endgroup$
              – rob
              Jan 3 at 0:59
















            • 4




              $begingroup$
              This question is not related to spin.
              $endgroup$
              – my2cts
              Jan 2 at 21:17






            • 2




              $begingroup$
              To the extent that all questions about electromagnetism involve spin, this answer is still quite wrong. The radiation from a rod antenna is linearly polarized. The photons which make up linearly-polarized radiation are in a superposition of left- and right-handed spin states. The sign of the more mobile charges in the antenna does not contribute.
              $endgroup$
              – rob
              Jan 3 at 0:59










            4




            4




            $begingroup$
            This question is not related to spin.
            $endgroup$
            – my2cts
            Jan 2 at 21:17




            $begingroup$
            This question is not related to spin.
            $endgroup$
            – my2cts
            Jan 2 at 21:17




            2




            2




            $begingroup$
            To the extent that all questions about electromagnetism involve spin, this answer is still quite wrong. The radiation from a rod antenna is linearly polarized. The photons which make up linearly-polarized radiation are in a superposition of left- and right-handed spin states. The sign of the more mobile charges in the antenna does not contribute.
            $endgroup$
            – rob
            Jan 3 at 0:59






            $begingroup$
            To the extent that all questions about electromagnetism involve spin, this answer is still quite wrong. The radiation from a rod antenna is linearly polarized. The photons which make up linearly-polarized radiation are in a superposition of left- and right-handed spin states. The sign of the more mobile charges in the antenna does not contribute.
            $endgroup$
            – rob
            Jan 3 at 0:59




















            draft saved

            draft discarded




















































            Thanks for contributing an answer to Physics Stack Exchange!


            • Please be sure to answer the question. Provide details and share your research!

            But avoid



            • Asking for help, clarification, or responding to other answers.

            • Making statements based on opinion; back them up with references or personal experience.


            Use MathJax to format equations. MathJax reference.


            To learn more, see our tips on writing great answers.




            draft saved


            draft discarded














            StackExchange.ready(
            function () {
            StackExchange.openid.initPostLogin('.new-post-login', 'https%3a%2f%2fphysics.stackexchange.com%2fquestions%2f451751%2fis-the-direction-of-the-electric-field-relative-to-the-magnetic-field-in-an-elec%23new-answer', 'question_page');
            }
            );

            Post as a guest















            Required, but never shown





















































            Required, but never shown














            Required, but never shown












            Required, but never shown







            Required, but never shown

































            Required, but never shown














            Required, but never shown












            Required, but never shown







            Required, but never shown







            Popular posts from this blog

            Can a sorcerer learn a 5th-level spell early by creating spell slots using the Font of Magic feature?

            Does disintegrating a polymorphed enemy still kill it after the 2018 errata?

            A Topological Invariant for $pi_3(U(n))$